What was price per share of the stock?

This topic has expert replies
Senior | Next Rank: 100 Posts
Posts: 85
Joined: Tue Mar 31, 2009 10:58 am

What was price per share of the stock?

by GmatGreen » Thu Mar 13, 2014 12:33 pm
If the total price of n equally priced shares of a certain stock was $12,000, what was the price per share of the stock?

1) If the price per share of the stock had been $1 more, the total price of the n shares would have been $300 more

2) If the price per share of the stock had been $2 less, the total price of the n shares would have been 5 percent less.

User avatar
GMAT Instructor
Posts: 1052
Joined: Fri May 21, 2010 1:30 am
Thanked: 335 times
Followed by:98 members

by Patrick_GMATFix » Thu Mar 13, 2014 12:47 pm
Great question. The answer is D. I go through the question in detail in the full solution below (taken from the GMATFix App).

Image

-Patrick
  • Ask me about tutoring.

User avatar
GMAT Instructor
Posts: 15539
Joined: Tue May 25, 2010 12:04 pm
Location: New York, NY
Thanked: 13060 times
Followed by:1906 members
GMAT Score:790

by GMATGuruNY » Fri Mar 14, 2014 11:03 pm
GmatGreen wrote:If the total price of n equally priced shares of a certain stock was $12,000, what was the price per share of the stock?

1) If the price per share of the stock had been $1 more, the total price of the n shares would have been $300 more

2) If the price per share of the stock had been $2 less, the total price of the n shares would have been 5 percent less.
n = the number of shares.
Let p = the price per share.

Statement 1: If the price per share of the stock had been $1 more, the total price of the n shares would have been $300 more.
n = (total price increase)/(increase per share) = 300/1 = 300.
Thus, p = 12000/n = 12000/300 = $40.
SUFFICIENT.

Statement 2: If the price per share of the stock had been $2 less, the total price of the n shares would have been 5 percent less.
If the total price decreases by 5%, then the value of p also decreases by 5%.
Statement 1 indicates that p=$40.
Decreasing p=40 by 5% would yield a decrease of $2 per share.
Thus, statement 2 implies the same information as statement 1: p=40.
SUFFICIENT.

The correct answer is D.
Private tutor exclusively for the GMAT and GRE, with over 20 years of experience.
Followed here and elsewhere by over 1900 test-takers.
I have worked with students based in the US, Australia, Taiwan, China, Tajikistan, Kuwait, Saudi Arabia -- a long list of countries.
My students have been admitted to HBS, CBS, Tuck, Yale, Stern, Fuqua -- a long list of top programs.

As a tutor, I don't simply teach you how I would approach problems.
I unlock the best way for YOU to solve problems.

For more information, please email me (Mitch Hunt) at [email protected].
Student Review #1
Student Review #2
Student Review #3

GMAT/MBA Expert

User avatar
Elite Legendary Member
Posts: 10392
Joined: Sun Jun 23, 2013 6:38 pm
Location: Palo Alto, CA
Thanked: 2867 times
Followed by:511 members
GMAT Score:800

by [email protected] » Sat Mar 15, 2014 1:57 pm
Hi GmatGreen,

This DS question is built around "system math."

The prompt tells us that N equally priced shares total $12,000. This can be written as:

N = Number of Shares
P = Price per share
NP = $12,000

This is 1 equation with 2 variables.

Fact 1: If the price per share was $1 MORE, then the total price would have been $300 more.

This Fact can be translated into:

N(P+1) = $12,300

Now we have a second equation, so we have "2 variables and 2 unique equations, so we CAN solve it."
Fact 1 is SUFFICIENT

Fact 2: If the price per share was $2 LESS, then the total price would be 5% less.

This Fact can be translated into:

N(P-2) = .95($12,000)

Again, we end up with a second equation, so this too is "2 variables and 2 unique equations, so we CAN solve it."
Fact 2 is SUFFICIENT

Final Answer: D

GMAT assassins aren't born, they're made,
Rich
Contact Rich at [email protected]
Image

User avatar
GMAT Instructor
Posts: 1248
Joined: Thu Mar 29, 2012 2:57 pm
Location: Everywhere
Thanked: 503 times
Followed by:192 members
GMAT Score:780

by Bill@VeritasPrep » Mon Mar 17, 2014 9:33 am
As Rich said, once you know that you CAN solve for a definite value of the variable, you are done. You don't actually have to solve for a price of $40; you just have to recognize that the pieces are in place to do so. An easy way to save time on DS.
Join Veritas Prep's 2010 Instructor of the Year, Matt Douglas for GMATT Mondays

Visit the Veritas Prep Blog

Try the FREE Veritas Prep Practice Test

GMAT/MBA Expert

User avatar
GMAT Instructor
Posts: 7243
Joined: Sat Apr 25, 2015 10:56 am
Location: Los Angeles, CA
Thanked: 43 times
Followed by:29 members
GmatGreen wrote:
Thu Mar 13, 2014 12:33 pm
If the total price of n equally priced shares of a certain stock was $12,000, what was the price per share of the stock?

1) If the price per share of the stock had been $1 more, the total price of the n shares would have been $300 more

2) If the price per share of the stock had been $2 less, the total price of the n shares would have been 5 percent less.
Solution:

Question Stem Analysis:


We need to determine the price per share of the stock, given that the total price of n shares of this stock was $12,000. Therefore, the price per share of the stock was 12,000/n dollars. In other words, if we can determine the value of n, then we can determine the price per share of the stock.

Statement One Alone:

We can create the equation:

(12,000/n + 1) * n = 12,000 + 300

12,000 + n = 12,300

n = 300

Since n = 300, the price per share of the stock was 12,000/300 = $40. Statement one alone is sufficient.

Statement Two Alone:

(12,000/n - 2) * n = 12,000 * 0.95

12,000 - 2n = 11,400

600 = 2n

300 = n

Once again, we see the price per share of the stock was $40. Statement two alone is sufficient.

Answer: D